[Toán 11]Bất đẳng thức

M

minhkhac_94

[TẶNG BẠN] TRỌN BỘ Bí kíp học tốt 08 môn
Chắc suất Đại học top - Giữ chỗ ngay!!

ĐĂNG BÀI NGAY để cùng trao đổi với các thành viên siêu nhiệt tình & dễ thương trên diễn đàn.

Bất đẳng thức thì trên diễn dàn mình cũng nhiều rồi (trong mục lớp 10) nhưng trong mục lớp 11 có ít topic về BĐT nên mình lập topic này để thảo luận về vẫn đề này
Trong này cũng có thể có các vấn đề ứng dụng của BĐT.
P/s Hi vọng các bài post được đánh đúng thứ tự của nó
Mở đầu bằng 1 bài
Bài 1(Marian Tetiva) Cho [tex]x,y,z[/tex] là các số thực không âm thoả mãn điều kiện
[tex]x^2+y^2+z^2+2xyz=1[/tex]
Chứng minh rằng:
[tex]1)xyz \le \frac{1}{8}[/tex]
[tex]2)x+y+z \le \frac{3}{2}[/tex]
[tex]3)xy+yz+zx \le \frac{3}{4} \le x^2+y^2+z^2[/tex]
[tex]4)xy+yz+zx \le \frac{1}{2} \le1+2xyz[/tex]
 
D

duynhan1

Bất đẳng thức thì trên diễn dàn mình cũng nhiều rồi (trong mục lớp 10) nhưng trong mục lớp 11 có ít topic về BĐT nên mình lập topic này để thảo luận về vẫn đề này
Trong này cũng có thể có các vấn đề ứng dụng của BĐT.
P/s Hi vọng các bài post được đánh đúng thứ tự của nó
Mở đầu bằng 1 bài
Bài 1(Marian Tetiva) Cho [tex]x,y,z[/tex] là các số thực không âm thoả mãn điều kiện
[tex]x^2+y^2+z^2+2xyz=1[/tex]
Chứng minh rằng:
[tex]1)xyz \le \frac{1}{8}[/tex]

[TEX] 1 - 2xyz = x^2 + y^2 + z^2 \ge 3\sqrt[3]{x^2y^2z^2} [/TEX]

[TEX] t = \sqrt[3]{xyz} [/TEX]

[TEX]\Rightarrow 1 - 2t^3 \ge 3t^2 [/TEX]

[TEX]\Leftrightarrow 2t^3 + 3t^2 - 1 \le 0 [/TEX]

[TEX]\Leftrightarrow ( t - \frac12 )( t+1 )^2 \le 0 [/TEX](*)

Do [TEX] t \ge 0 [/TEX] nên (*) [TEX]\Leftrightarrow t \le \frac12 \Leftrightarrow xyz \le \frac18[/TEX]
 
M

minhkhac_94

Bài 1 Nếu đặt [tex]x=sin(\frac{A}{2}[/tex] .... thì tất cả phần trên dễ rồi
Bài 2Cho các số ko âm [tex]x,y,z[/tex]
[tex]\frac{ab}{a+b+2c}+\frac{bc}{b+c+2a}+\frac{ca}{c+a+2b} \le \frac{1}{4}(a+b+c}[/tex]
 
T

thesecond_jerusalem

ủng hộ mấy bài IMO nè :)) toàn dùng AM-GM hết nhé

1)Cho các số thực dương a,b,c .cmr:
[tex]\frac{1}{a^3+b^3+abc}+\frac{1}{b^3+c^3+abc}+\frac{1}{c^3+a^3+abc} \leq \frac{1}{abc} [/tex]
_________________________ ___USA IMO 1998____________________________

2)Cho các số thực dương x,y,z thỏa ĐK:[tex]x^3+y^3+z^3=3[/tex].CMR:

[tex]\frac{xy}{z}+\frac{yz}{x}+\frac{xz}{y}\geq3[/tex]
______________________________France Pre-IMO 2005________________________

 
M

minhkhac_94

ủng hộ mấy bài IMO nè :)) toàn dùng AM-GM hết nhé

[/\b]Bài 3[/b])Cho các số thực dương a,b,c .cmr:
[tex]\frac{1}{a^3+b^3+abc}+\frac{1}{b^3+c^3+abc}+\frac{1}{c^3+a^3+abc} \leq \frac{1}{abc} [/tex]
_________________________ ___USA IMO 1998____________________________

Bài 4)Cho các số thực dương x,y,z thỏa ĐK:[tex]x^3+y^3+z^3=3[/tex].CMR:

[tex]\frac{xy}{z}+\frac{yz}{x}+\frac{xz}{y}\geq3[/tex]
______________________________France Pre-IMO 2005________________________


Lần sau post đúng tên bài nhé :D

Bài 3
Ta có [tex]a^3+b^3\geq ab(a+b)<=>(a-b)^2(a+b) \geq 0 [/tex](đúng)
Do dó ta có [tex]\frac{1}{a^3+b^3+abc}+\frac{1}{b^3+c^3+abc}+\frac{1}{c^3+a^3+abc} \leq \sum{\frac{1}{ab(a+b)+abc} [/tex]
[tex]\sum{\frac{1}{ab(a+b)+abc}=\frac{1}{abc}[/tex]
Ta có đpcm
 
H

herrycuong_boy94

latex.php

gif.latex

gif.latex
 
G

giaosu_fanting_thientai

Bài IMO thứ 2 sai đề kìa:p

1. Cm [TEX](1+\frac{1}{n+1})^{n+1} > (1+ \frac {1}{n} )^n[/TEX] với mọi n>0

2. mọi số tự nhiên n \geq2. Cm:
[TEX]\sqrt[n+1]{\frac {n+1}{2}} < \frac {1}{1^2} +\frac{1}{2^2} +....+\frac {1}{n^2}[/TEX]

2 bài nì mod vodichhocmai nợ mình:) Hứa lên lớp 11 mờ k bjk mod còn nhớ k:D
Nhưng vẫn làm được 2 bài nèy theo kiểu lớp "10" ;)
 
P

phamduyquoc0906

Bài IMO thứ 2 sai đề kìa

1. Cm [TEX](1+\frac{1}{n+1})^{n+1} > (1+ \frac {1}{n} )^n[/TEX] với mọi n>0

2. mọi số tự nhiên n 2. Cm:
[TEX]\sqrt[n+1]{\frac {n+1}{2}} < \frac {1}{1^2} +\frac{1}{2^2} +....+\frac {1}{n^2}[/TEX]

2 bài nì mod vodichhocmai nợ mình Hứa lên lớp 11 mờ k bjk mod còn nhớ k
Nhưng vẫn làm được 2 bài nèy theo kiểu lớp "10"

*Bạn giải được rồi mà nhờ anh vodichhocmai làm gì nữa nhỉ
*Giải bằng phương pháp hàm số
[TEX]f(t)=(1+\frac{1}{t})^t\ \ \ \ (t>0)[/TEX]
[TEX]lnf(t)=t.ln(1+\frac{1}{t})\Rightarrow{\frac{f^'(t)}{f(t)}=ln(1+\frac{1}{t})-\frac{1}{t+1}=g(t)[/TEX]

[TEX]g^'(t)=-\frac{1}{t(t+1)}+\frac{1}{(t+1)^2}=-\frac{1}{t(t+1)^2}<0\ \ \ \forall{t>0[/TEX][TEX]\Rightarrow{g(t):NB[/TEX]
[TEX]g(0)=+\infty\ ,\ g(+\infty)=0\ \ (ghi---lim--)\ ,\ BBT\Rightarrow{g(t)>0\ \ \ \forall{t>0[/TEX]

[TEX]\Rightarrow{f^'(t)=f(t).g(t)>0\ \ \ \forall{t>0[/TEX][TEX]\Rightarrow{f(t):DB[/TEX]

[TEX]f(n+1)>f(n)\Leftrightarrow{(1+\frac{1}{n+1})^{n+1} > (1+ \frac {1}{n} )^n[/TEX]
 
M

minhkhac_94

Bài 7 Cho [tex]x,y,z[/tex] ko âm thỏa mãn [tex]x+y+z=1[/tex]. Chứng minh rằng :
[tex]\sum{\frac{1}{1-xy}} \le \frac{27}{8}[/tex]
Bài 8: Cho [tex]a,b,c [/tex]là số thực dương. Chứng minh rằng :
[tex]\frac{(2a+b+c)^2}{2a^2+(b+c)^2}+\frac{(2b+a+c)^2}{2b^2+(a+c)^2}+\frac{(2c+a+b)^2}{2c^2+(a+b)^2}} \le 8[/tex]
 
D

duynhan1

Bài 7 Cho [tex]x,y,z[/tex] ko âm thỏa mãn [tex]x+y+z=1[/tex]. Chứng minh rằng :
[tex]\sum{\frac{1}{1-xy}} \le \frac{27}{8}[/tex]

[TEX]\Leftrightarrow 8( 3 - 2 ( xy + yz + zx ) + xyz(x+y+z) ) \le 27( 1 - ( xy + yz + zx) +xyz(x+y+z) - x^2y^2z^2 ) [/TEX]

[TEX]\Leftrightarrow 24 - 16 q + 8r \le 27 - 27 q + 27r - 27r^2 [/TEX]

[TEX]\Leftrightarrow 3 -11 q + 9r - 27r^2 \ge 0 [/TEX]

[TEX]\Leftrightarrow (1 - 3q) + r( 1 - 27r) + 2( 9r - 4 q + 1 ) \ge 0 [/TEX]

Điều này luôn đúng do :

[TEX]\left{ q \le \frac13p^2 = \frac13 \\ r \le \frac{1}{27} p^3 = \frac{1}{27} \\ 9r \ge p( 4q-p^2) = 4q - 1 [/TEX]
 
M

minhkhac_94

Bài 9 Tìm giá trị lớn nhất của hàm số
gif.latex
 
Last edited by a moderator:
M

minhkhac_94

Nhanh phết ! sửa lại đề rồi sorry nhé :D ______________
Thế này có khác j đâu :))
Bài 10: Cho
gif.latex
thuộc đoạn
gif.latex
thỏa mãn
gif.latex
Tìm GTNN và GTLN của
gif.latex
 
Last edited by a moderator:
L

letrang3003

Nhanh phết ! sửa lại đề rồi sorry nhé :D ______________
Thế này có khác j đâu :))
Bài 10: Cho
gif.latex
thuộc đoạn
gif.latex
thỏa mãn
gif.latex
Tìm GTNN và GTLN của
gif.latex

[TEX]\cos \frac{9}{4} =\cos\bigg(a+b+c\bigg)^2 \le \cos (a^2+b^2+c^2) \le \cos \bigg(\frac{(a+b+c)^2}{3}\bigg)=\cos \frac{3}{4}[/TEX]

ta thấy[TEX] \frac{9}{4}[/TEX] và [TEX]\frac{3}{4}[/TEX] đều trong khoảng[TEX] [0,\pi][/TEX] nên có đpcm:

hình như thừa đk [TEX]x,y.z \le 1[/TEX] đó anh .
 
Last edited by a moderator:
L

legendismine

Bất đẳng thức thì trên diễn dàn mình cũng nhiều rồi (trong mục lớp 10) nhưng trong mục lớp 11 có ít topic về BĐT nên mình lập topic này để thảo luận về vẫn đề này
Trong này cũng có thể có các vấn đề ứng dụng của BĐT.
P/s Hi vọng các bài post được đánh đúng thứ tự của nó
Mở đầu bằng 1 bài
Bài 1(Marian Tetiva) Cho [tex]x,y,z[/tex] là các số thực không âm thoả mãn điều kiện
[tex]x^2+y^2+z^2+2xyz=1[/tex]
Chứng minh rằng:
[tex]1)xyz \le \frac{1}{8}[/tex]
[tex]2)x+y+z \le \frac{3}{2}[/tex]
[tex]3)xy+yz+zx \le \frac{3}{4} \le x^2+y^2+z^2[/tex]
[tex]4)xy+yz+zx \le \frac{1}{2} \le1+2xyz[/tex]
dat s=x+y+z,ta có
[tex]s^2-2s+1\le \2(\frac{1-x+1-y+1-z}{3})^3=2(\frac {3-s}{3})^3[/tex]
[tex](2s-3)(s+3)^2\le 0[/tex]

[tex]xy+yz+xz\le \frac {3}{4} , x^2+y^2+z^2=1-2xyz=\frac {3}{4}[/tex]
 
L

legendismine

ủng hộ mấy bài IMO nè :)) toàn dùng AM-GM hết nhé

1)Cho các số thực dương a,b,c .cmr:
[tex]\frac{1}{a^3+b^3+abc}+\frac{1}{b^3+c^3+abc}+\frac{1}{c^3+a^3+abc} \leq \frac{1}{abc} [/tex]
_________________________ ___USA IMO 1998____________________________

2)Cho các số thực dương x,y,z thỏa ĐK:[tex]x^3+y^3+z^3=3[/tex].CMR:

[tex]\frac{xy}{z}+\frac{yz}{x}+\frac{xz}{y}\geq3[/tex]
______________________________France Pre-IMO 2005________________________

2>
[tex]\sum_{cyc}\frac {x^2y^2}{z^2}+2(x^2+y^2+z^2)\ge 3(x^2+y^2+z^2)[/tex]
 
Top Bottom